You are on page 1of 31

RPEB_Simulated Test Paper_01_2013 - Attempt 1 Top of Form Question 1 Marks: 1 _________ is not a step involved in Financial Planning.

Choose one answer. a. Data Gathering b. Periodic Review of the Plan c. Plan Preparation d. Advising a client on how to evade tax Question 2 Marks: 1 Part 1: The core financial planning competency under the Analysis function relates to _________. Choose one answer. a. the financial planning professional synthesizes the information to develop a Financial Plan b. the financial planning professional synthesizes the information to evaluate strategies to create a Financial Plan c. the financial planning professional identifies and considers issues, performs financial analysis d. the financial planning professional collects the information required to develop a financial plan Question 3 Marks: 1 The number of members covered in a group insurance plan_______. Choose one answer. a. Are fixed

b. Remain the same c. Fluctuate d. None of the above Question 4 Marks: 1 If you want to tweak the cover on a policy and customize then this will not be possible in ________. Choose one answer. a. Group Insurance b. Individual insurance c. Term Insurance d. Unit linked policy Question 5 Marks: 1 Which one of the following statements is NOT true for a defined-benefit plan. (i) It favors older participants. (ii) Contributions are annual & arbitrary. (iii) It requires an actuary. Choose one answer. a. (i) only b. (ii) only c. (iii) only d. None of the statements Question 6 Marks: 1

For defined benefit plans, which of the following changes in actuarial assumption would increase plan costs to an employee? 1. Early retirement without a reduction in benefits. 2. Longer life expectancy predicted 3. An increase in inflation expectations regarding labor costs. Choose one answer. a. 1 & 2 only b. 1 only c. 1, 2 & 3 d. 2 & 3 only Question 7 Marks: 1 Business Risk is associated with Choose one answer. a. Economy as a whole b. International Market c. Nature of Enterprise d. None of the above Question 8 Marks: 1 What options does an employer have for superannuation arrangement? Option (A) - Payment by employer Option (B) - Funding through a trust Choose one answer. a. Only (A)

b. Only (B) c. Neither (A) nor (B) d. Both (A) & (B) Question 9 Marks: 1 Under EPF scheme, monthly pension after retirement is decided on _________. Choose one answer. a. eligible service and pensionable salary b. pensionable salary & contributory service). c. pensionable service and pensionable salary d. None of the above. Question 10 Marks: 1 Which of the following is a defined contribution plan? Choose one answer. a. Leave salary b. Gratuity c. Unrecognized Provident Fund d. Voluntary Retirement Scheme Question 11 Marks: 1 The Employee's Deposit Linked Insurance is applicable to all establishments covered under: Choose one answer.

a. The Employee's Provident Fund and Miscellaneous Provision Act b. Payment of Gratuity Act- 1972 c. Workmens Compensation Act- 1923 d. Provident Funds Act- 1925 Question 12 Marks: 1 As an employee Mohan has come to you with his question on superannuation plans/ Annuity Plans to get more educated with current scenario. The term 'Employee benefits' includes contributions in which of the following categories of schemes? Choose one answer. a. Non occupation disability Insurance b. Unemployment compensation Insurance c. Social security Scheme d. Group Insurance Scheme Question 13 Marks: 2 Which of the following statement/s is/are correct? (A) Any salary surrendered by the employee to the central Government under the voluntary Surrender of Salaries Act, 1961, will not be included while calculating his taxable income if he is a Government employee. (B) Any salary surrendered by the employee to the central Government under the voluntary Surrender of Salaries Act, 1961, will not be included while calculating his taxable income if he is a private/public sector employee. Choose one answer. a. only (B) is correct b. only (A) is correct.

c. Both (A) and (B) are wrong. d. Both (A) and (B) are correct Question 14 Marks: 2 Ashok receives leave encasement of Rs. 2,00,000 on account of accumulated leave of 240 days. He has completed 24 years of service and his last 10 months average monthly salary is Rs. 10,000. What is the taxable portion of leave encasement for Ashok if he was entitled to 40 days leave for every year of service? Choose one answer. a. Rs. 1,00,000 b. Rs. 2,00,000 c. Nil. d. Rs. 2,50,000 Question 15 Marks: 2 An insurance firm has to create and sell a policy to cover the risk of partial disability to workmen in a chemical factory. Which of the following is not a relevant factor in pricing the policy? Choose one answer. a. The ability of the insurance firm to deploy the premium and earn positive investment returns on the funds b. The ability of the workmen to bear the cost of the policy c. The extent of probable disability created due to the work environment and its impact on workmen.s earning capability. d. The willingness of the management of the firm to bear part of the costs of the premium Question 16 Marks: 2

Amit is eligible for gratuity as per the provisions of the Payment of Gratuity Act, 1972. Amits employer has agreed to pay him a gratuity amount that is higher than what is stipulated under the Act. Amit accordingly would receive a final settlement of Rs. 12,00,000/- on retirement. What is the tax status of this amount? Choose one answer. a. The gratuity paid is exempt from Income Tax only to the extent of Rs.10,00,000/-. b. The amount of gratuity payable to him cannot exceed Rs. 3,50,000/-. c. There is no specific tax exemption for gratuity, in the Income Tax Act. d. The Income Tax Act will only allow a maximum exemption upto 15 days wages per completed year of service. The rest is taxable. Question 17 Marks: 2 Which of the following Financial Planner Professional Skills would not be categorized under the segment Cognitive? Choose one answer. a. Uses logic and reasoning to consider the strengths and weaknesses of potential courses of action . b. Arrives at informed decisions when faced with incomplete or inconsistent information c. Deals effectively with objections and complaints d. Demonstrates capacity to adapt thinking and behaviors Question 18 Marks: 1 The following are critical parameter/s for a companys stock to be included in an exchanges equity index: Choose one answer. a. Average price quoted in the last six months

b. Market capitalization and liquidity c. Industry leadership in terms of sales volume d. Market trading span of at least three years Question 19 Marks: 1 You can find Basic liquid ratio with which two variables: Choose one answer. a. Fixed assests and Monthly Expenses b. Liquid Assests & Monthly Expenses c. All Assests + Monthly expenses d. None of the above Question 20 Marks: 1 Which one of the following rightly describe prospect theory? Choose one answer. a. A theroy that describes how individuals in a group can act together without planned direction b. A theory that people value gains and losses differently c. A therory that investors base their decisions on irrelevant figures and statistics d. None of the above Question 21 Marks: 1 Calculate the GDP of India with the given figures: Consumption: $ 300 billion Investments: $ 400 billion Government spending: $ 300 billion

Exports: $ 150 billion Imports: $ 200 billion Choose one answer. a. $1,000 billion b. $650 c. $1,050 d. $950 Question 22 Marks: 1 The process of retirement planning would generally not involve ________. Choose one answer. a. concentrating on maximizing returns on Present / Past Investments b. projecting individual needs and goals into the future and making sound financial plan c. making a plan for management & disposition of assets at death d. planning for staying physically healthy & making necessary psychological adjustment and to plan for housing and leisure / work Question 23 Marks: 1 Changing the weights of assets in the portfolio at regular intervals is called _____. Choose one answer. a. Diversifying b. Rebalancing c. Concentrating d. None of the above

Question 24 Marks: 1 One of the biggest reasons to undertake portfolio evaluation is _________. Choose one answer. a. poor planning b. no analysis c. changing needs of the individual and financial goals d. young age of the person for whom the plan is made Question 25 Marks: 2 If the post tax rate of return on an investment is 8% and the inflation rate is 5% the real rate of return is_______________. Choose one answer. a. 3.5% b. 3.0% c. 2.86% d. -3.0% Question 26 Marks: 2 If the inflation rate is 4.9% and tax rate is 30% the required rate of return to maintain the value of an investment will be ___________. Choose one answer. a. 8% b. 9%

c. 7% d. 10% Question 27 Marks: 3 Sahil, age 43, can refinance Rs. 114042 at a 20-year rate for 7% and will incur closing cost of 3% of the mortgage amount to be financed in the new mortgage balance. What will be his new EMI on the mortgage under the circumstances to achieve his objective of no debt at retirement (age 60)? Choose one answer. a. Rs. 781.49 b. Rs. 957.56 c. Rs. 980.57 d. Rs. 986.29 Question 28 Marks: 3 Sumeet, aged 25, plans to retire at age 55. His life expectancy is 75. His current annual expenditure is Rs. 250000. He estimates no reduction in his expenses post-retirement. If interest rate is expected to be 8.5% and inflation is 5% p.a. estimate how much will he have to save per annum in order to achieve his target, provided he does not wish to leave an estate? Choose one answer. a. Rs. 129696 b. Rs. 125054 c. Rs. 117154 d. Rs. 120963 Question 29 Marks: 3

Aman wants to retire after 20 years. Aman needs a retirement corpus of 3 crores. Aman wants to invests in such a way that his contributions keep on increasing by 20% every year. Expected ROI is 15%. What amount Aman will invest in first year to achieve his goal? Choose one answer. a. 59336 b. 34030 c. 58922 d. 70393 Question 30 Marks: 3 13. A 45-year old man spends Rs. 6 lakh p.a., almost the amount he earns, to maintain his family. He expects his expenses to rise by 6% p.a. He has got saving of Rs.1000,000 for retirement. He has a second house which he wants to rent at Rs. 15,000 p.m. immediately, the rent expected to increase by 6 % p.a. You advise him to create a corpus by his age of 60 by investing the rent received & his saving in an instrument yielding 9% p.a. at the end of every year. You estimate the number of years the accumulated corpus would last _____ Choose one answer. a. 28 years b. 13 years c. 12 years d. 8 years Question 31 Marks: 3 Mr. Rajesh who is 35 years old spends annually Rs. 6 lakh towards his household expenses. He expects to retire at 60 years. During this period inflation is expected to be on an average 6% p.a. He wants to cover 25 years living expenses for self and spouse & wants to give Rs.75 lac ( then value) in charity at his life expectancy (Age 85). He expects to generate a return of 8% from his accumulated corpus, what corpus should he target for a comfortable retirement & charity?

Choose one answer. a. 5.2 crore b. 5.3 crore c. 2.9 crore d. 3.0 crore Question 32 Marks: 3 Mr.Modi has recently purchased a house worth Rs. 40 lakh for self-occupation by taking a home loan of Rs. 25 lakh at 10 % p.a. rate of interest. The tenure of loan is 15 years. He has Rs. 12 lakh financial assets (50% in equity and 50% in debt) at present. He is expected to save monthly Rs. 20000 which he invested in equity. Return from equity is expected @12% pa and debt @ 8.5% pa What would be his net worth five years from now? The value of the house which is for consumption purposes is not considered in the net worth so arrived. Choose one answer. a. 26 lakh b. 15.5 lakh c. 24.5 lakh d. 35.8 lakh Question 33 Marks: 4 Puskar , age 40, spends Rs. 3 lakh p.a. . He plans to retire at age 60 and expects to live till 75 years. His annual income is Rs. 7 lac pa. The basic inflation at 7% p.a. and lifestyle inflation at 1.75% p.a. are expected in the pre-retirement period. He starts investing 20% of his income in a 10% p.a. return instrument with immediate effect. He assumes his salary will increase every year by 8%. If the expenses post-retirement are curtailed by 20%, what maximum inflation would sustain his corpus till he survives, if the corpus is invested at 8% p.a.? Choose one answer.

a. 5% b. 4% c. 2.9% d. 5.5% Question 34 Marks: 1 Interest rate on Post Office Savings Bank is payable__________. Choose one answer. a. monthly b. quarterly c. half yearly d. Yearly Question 35 Marks: 1 The investment limit in PO MIS, which can save tax under section 80C of the Income Tax Act, 1961 to a resident individual is: Choose one answer. a. Min Rs. 6000/- and no upper limit b. Min Rs. 6000/- Max 6 lakh per assessee c. Min Rs. 6000/- Max 3 lakh per assessee d. None of the above Question 36 Marks: 1 Investment in Senior Citizens Pension Plan qualifies for deduction u/s 80C of IT Act.

Choose one answer. a. True b. False c. Only upto Rs. 70,000 d. Upto Rs.100,000 Question 37 Marks: 1 The Real Rate of Return necessarily takes into account which of the following : Choose one answer. a. Compounding b. Inflation c. Discounting d. Present Value Question 38 Marks: 2 Which of the following financial transactions / events would affect net worth of your client? (A) Repayment of a loan using fund from a savings account. (B) Purchase of car which is 75% financed with 25% down payment. (C) The Nifty is appreciating, and the client is holding Nifty Indexed Mutual Fund. (D) Interest rate increases and the client holds substantial bond portfolio Choose one answer. a. (C) & (D) b. (B) & (C) c. (A), (C) & (D) d. All of the above

Question 39 Marks: 2 Mrs. & Mr. Arora are aged 55 and 58 years respectively. Both expect to work till they turn 65. Their only goal is to fund their retirement. Which of the following is likely to be an appropriate asset allocation strategy for them? Choose one answer. a. 10% sectoral equity, 20% diversified equity, 30% long-term debt, and 40% medium term debt b. 20% Sectoral equity, 60% diversified equity, 20% long-term debt c. 30% Sectoral equity, 30% diversified equity, 40% cash/ liquid investments d. 80% long-term debt, 20% medium term debt Question 40 Marks: 3 Aditi is 30 years old. She deposits 25000 at the beginning of each year in deferred annuity scheme as a part of her retirement planning. How much will be in her account after 25 years if her savings earn 10% in first 10 years, then 9% in next 10 years and 8% in last 5 years? Choose one answer. a. Rs. 2474985 b. Rs. 20,90,000 c. Rs. 22,91,000 d. Rs. 24,97,000 Question 41 Marks: 3 Mr. Rajans investment portfolio comprises Rs.2 lakh in equity, Rs.5 lakh in debt and Rs. 1 lakh in his bank current account. Over one year the returns on equity and debt are 5% and 12%. At the end of the year to maintain his current asset allocation, he needs to _____________ Choose one answer.

a. Do nothing. b. He needs to move Rs, 10000/- from equity and Rs. 60000/- from debt to cash. c. He needs move Rs.7500/- to equity from debt and Rs. 8750/-to cash from debt d. He needs to invest Rs. 70000/- in debt and equity. Question 42 Marks: 3 Rajiv got offer of 4 products 1. giving returns of 9% pa compounding monthly, 2. give returns of 9.5% compounding semi annually 3. 9.25% pa compounding semi annually 4. giving returns of 9.6% pa, which product is giving him highest returns? Choose one answer. a. Product 1 b. Product 2 c. Product 3 d. Product 4 Question 43 Marks: 3 Ram is investing Rs.10,000 every month in an annuity plan. He starts investment from the age of 35 and will continue to invest till age 60. How much annuity he will get at the beginning of every month from the age 60 to age 80? Return of return in accumulation period is 9%pa and in retirement period is 8%pa. Choose one answer. a. 91,934 b. 93,982 c. 86,706 d. 87,264 Question 44

Marks: 3 Ramesh wants to calculate his monthly expenses after 20 years from now. His current monthly expense is Rs.25,000. Inflation for 5 years will be 7%. Then it will reduce to 6% for next 5 years . For the last 10 years inflation rate will be 5% pa. Choose one answer. a. 76,443 b. 59,887 c. 72,895 d. 59,121 Question 45 Marks: 3 Aishwariya, age 30 , plans to retire at 55. She has been investing Rs. 60,000 every year in a Mutual fund with expected return of 11% since last 4 years and will continue to do till her retirement. Her current expenses are Rs. 30,000 per month. She wants to cover 30 years post-retirement expenses inflationadjusted by a suitable annuity. Average inflation is considered at 7% p.a. and the annuity is expected to yield 9% p.a. To build her retirement corpus she can incrementally invest Rs. 2 lakh annually, you optimize average rate of return to achieve this goal. The same is _____. Choose one answer. a. 11.1% b. 19.65% c. 15.41% d. 12.61% Question 46 Marks: 4 Rajiv, a Medical profesional, retires today has two fixed life annuities, one provided by his invested pension plan at Rs. 15,000 per month and the other provided by his employer at Rs. 30,000 per month. He also has Rs.11 lac in a Mutual fund created for retirement. He has a second house which currently is let out at Rs. 2 lakh p.a. rental. The rentals are expected to increase at 7 % p.a. compounded. He

currently spends Rs. 5 lac every year which will rise annually at inflation of 6 % p.a. If he invests excess amount at the end of every year along with Rs.11 lac in Mutual fund in an instrument of return 9 % p.a. and utilizes this fund in case of shortfall in funding household expenses, what could be the approximate size of this fund 25 years after retirement? Choose one answer. a. 17 lakh b. 55 lakh c. 69.6 lakh d. 1.42 crore Question 47 Marks: 4 Mr. A purchased a flat worth Rs. 60 lakh in January 2008 by availing a housing loan of Rs. 45 lakh for tenure 20 years at the rate of 9.5% p.a. The value of his flat as in January 2013 has appreciated to Rs. 1 crore. What approximate value of home equity can he consider in his flat towards his unencumbered interest after also setting aside 10% of the appreciation value towards taxes and other costs to be discharged on selling the unit? Choose one answer. a. 52.3 lakh b. 91.1 lakh c. 96.0 lakh d. 55.8 lakh Question 48 Marks: 4 Nirav, aged 30, wants to retire at 45. He wants to maintain his present living standard. He spends Rs. 3,25,000 a year. He is expected to live upto 85. Inflation is to be assumed at 4% p.a and expected returns are 7% p.a. What is the nest egg required by Nirav at his age 45 and what amount should he save every year end to meet his plan? His present investments are Rs 10,00,000. Choose one answer.

a. Nest egg and savings required will be Rs. 1,41,82,808 and Rs. 4,54,604 respectively. b. Nest egg and savings required will be Rs. 1,47,73,065 and Rs. 7,40,530 respectively c. Nest egg and savings required will be Rs. 1,27,73,065 and Rs. 4,38,300 respectively d. Nest egg and savings required will be Rs. 2,51,00,065 and Rs. 5, 41,093 respectively Question 49 Marks: 4 Ahmeds current annual expenditure is Rs.100000/-. He is 30 years old and expects to retire at age 55. His annual expenses are estimated to rise by 6% p.a. and his life expectancy is 75 years. His post retirement annual expenses are estimated to be 80% of his pre-retirement expenses. What will be his expenses on the first year of his retirement? Choose one answer. a. Rs. 3,47,425/b. Rs. 3,43,350/c. Rs. 4,29,187/d. Rs. 4,02,350/Question 50 Marks: 4 Vikas , who wants to retire at age 60 , has invested annually Rs. 1.5 lakh towards his retirement in an aggressive fund from year his age of 40 on wards assuming the rate of return of 14% pa. But due to economic slowdown his fund could able to generate only the return of 3% pa in the first 5 years. He can direct a higher amount towards retirement fund that he would have achieved with 14% return in 20 years. He wants to know how much additional fund he has to invest every year from the age of 45 for next 15 years, if the expected rate of return from the fund is 10% pa. Choose one answer. a. 2.75 lakh b. 4.25 lakh

c. 4.45 lakh d. 2.95 lakh Question 51 Marks: 4 Ms.Rekha is 45 years old and plans to retire at 50. Her life expectancy is 70 years. Ms. Sushma, her Financial Planner, estimates that her client will require Rs.45,000/- in the first month after retirement. This expenses will continue to grow because of inflation. Inflation rate is 4% p.a. and the rate of return is 6% p.a. What will be the savings per year required in order to meet this? Choose one answer. a. Rs. 15,90,000/- (Approx) b. Rs. 15,00,000/- (Approx) c. Rs. 16,90,000/- (Approx) d. Rs. 12,40,000/- (Approx) Question 52 Marks: 1 Which of the following is in adherence with the professional responsibility of a CFP professional towards the client? Choose one answer. a. Competing with professionals in specialized services to internalize most deliverables b. Referring the client to other professionals for certain duration with transfer of liability c. Having legitimate difference of opinion on an issue from fellow professionals and the client d. Managing one's own prejudices and desires to achieve a proper balance of interests Question 53 Marks: 1 Which of the following is a correct interpretation of the Rules of Conduct pertaining to the Ethic of Confidentiality?

Choose one answer. a. A Member must when requested by the client, provide to a person authorized by the client, all original documents prepared or received by the Member in undertaking the advisory task. b. A Member owes to the Member.s partners or co-owners a responsibility to act in good faith (expectations of confidentiality) only while in business together, not thereafter c. The Member shall maintain the same standards of confidentiality to employers as to clients d. Under no circumstance, will any Member divulge any information or knowledge regarding the FPSB India or its members that they may know or be exposed to Question 54 Marks: 1 Kulkarni is running an unrecognized Provident Fund, because he has a smaller number of employees in his firm. Which of the following is true about the tax status of such a fund? Choose one answer. a. Only the employees. contribution is eligible for a rebate under Section 80C. b. Kulkarni.s contribution as the employer is exempt from tax in his books. c. The interest received on such unrecognised PFs is taxable every year. d. Accumulated employees contribution is taxable at the time of retirement. Question 55 Marks: 1 Which amongst the following is true about the interest received from Recognized Provident Fund (RPF)? Choose one answer. a. Not treated as income in the year of credit. b. Not treated as income if the rate is equal to/less than 8.5%. Rate over and above this is taxable c. Fully exempt from tax. d. None of the above

Question 56 Marks: 1 NPS is applicable to all Citizens of India and NRI's falling between the age group of Choose one answer. a. 18 & 55 b. 18 & 60 c. 21 & 60 d. 21 & 55 Question 57 Marks: 1 An annuitant under an Annuity Policy of Insurance company is the following person: Choose one answer. a. A person who receives annuity b. A nominee c. An assignee. d. None of the above. Question 58 Marks: 2 Raykar is an accomplished Financial Planner and is also an expert on derivatives and high yielding bonds. He understands client requirements well and is able to come up with appropriate portfolio restructuring ideas for clients. He believes in quickly moving clients from one investment to another through a dynamic process of research and recommendations. What according to the Rules relating to the Code of Ethics is the most applicable in this case? Choose one answer. a. He does not violate the Rules if he explains to the client the reasons and is able to show that the moves are appropriate to the client.

b. He does not violate the Rules since he conducts and has access to research and advises on products relevant to clients based on an understanding of their requirements. c. He does not violate the Rules since he is an acknowledged expert and knows what is best for his clients. d. He violates the Rules as it amounts to active churning of client portfolios. Question 59 Marks: 2 An eligibility criterion for withdrawal from a Provident fund for purchase of a house is ___________. Choose one answer. a. membership of the fund for 5 years, minimum balance in members a/c Rs. 1,000, purchase should be in favour of the member or member and spouse. b. membership of the fund for 10 years, minimum balance in members a/c Rs. 1,000, purchase should be in favour of the member or member and spouse. c. membership of the fund for 5 years, minimum balance in members a/c Rs. 5,000, purchase should be in favour of the member or member and spouse. d. membership of the fund for 5 years, minimum balance in members a/c Rs. 1,000, purchase should be in favour of the member or spouse. Question 60 Marks: 2 The maximum advance a member can take from his/her Provident Fund, for major illness involving stay in hospital for at least 1 month, is ______. Choose one answer. a. 12 months wage (basic + DA) b. 24 months wage (basic + DA) c. 6 months wage (basic + DA) d. 36 months wage (basic + DA) Question 61

Marks: 3 Omprakash read a draft offer document that PFRDA has come out with a New Pension Scheme (NPS) for all citizens of India. He is also thinking to invest in NPS but he is confused with regards to the withdrawal provisions of the scheme in Tier-I. You are required to provide him with the correct details of the withdrawal. i. If he exits before 60 years of age, he will have to invest at least 20% of the pension wealth to purchase a life annuity and the rest 80% of pension wealth may be withdrawn as a lump sum. ii. If he exits on attaining 60 years of age, he will have to invest at least 40% of the pension wealth to purchase a life annuity and the rest 60% of pension wealth may be withdrawn as a lump sum or in a phased manner between ages 60 and 70 years. iii. If he exits before 60 years, he will have to invest at least 80% of the pension wealth to purchase a life annuity and the rest 20% of pension wealth may be withdrawn as a lump sum. iv. If he exits on attaining 60 years of age, he will have to invest at least 60% of the pension wealth to purchase a life annuity and the rest 40% of pension wealth may be withdrawn as a lump sum or in phased manner between ages 60 and 70 years. Choose one answer. a. i & iv b. i & ii c. ii & iii d. iii & iv Question 62 Marks: 3 A person invested Rs. 45 lakh in a 30-year fixed monthly annuity providing a yield of 9 % p.a. How much less amount he will get every year if the rate of return reduced to 8.5% pa. Choose one answer. a. 1450 b. 1580 c. 13455 d. 13505

Question 63 Marks: 4 Pranay , age 60 ,will retire in the next week. He has accumulated Rs. 60 lakh towards retirement funds. He purchases an immediate annuity for a total term of 25 years, a fixed monthly amount for the initial period of 10 years and a provision to double the monthly amount in the second 10-year period and triple ( three times of first annuity) the amount in the last 5 -year period . If the minimum yield guaranteed in the annuity is 9% p.a., what monthly amount he is expected to receive in the last 5-year period? Choose one answer. a. 0.68 lakh b. 1.035 lakh c. 0.7 lakh d. 1.02 lakh Question 64 Marks: 4 A is getting a salary of 500000 per annum. He invests 20 % of the salary every year for his retirement for 15 years. His salary is expected to increase by 5 % every year and his expected return is 12 %.what is the fund value he will have at the end of 15 years ? A. Assume money saved at the end of year. Choose one answer. a. 48.49 lakh b. 54.31 lakh c. 37.27 lakh d. 41.75 lakh Question 65 Marks: 4 Suppose you have been investing ` 70,000 in the PPF account since April 2008, (you opened the account then) and have been investing the said amount till April 2011, (for the year 2011 you invested only ` 70,000 and the rate increased from 8% to 8.6% in December 2011), from April 2012 the rate has

increased to 8.8% and it will be the same till end. If you continue to invest Rs.1,00,000 every year from April 2012, what will you have on maturity? Choose one answer. a. 29.29 lakh b. 31.04 lakh c. 28.95 lakh d. 30.33 lakh Question 66 Marks: 1 An establishment with 100 employees, notified by Central Government and also that which is a factory engaged in a specified industry, would be covered under Employees Provident fund and Miscellaneous provision Act, 1952 if the number of employees, getting salary of Rs.6500 or less, is/are _____________. Choose one answer. a. 20 or more b. It will be covered even if the income of all employees is above Rs. 6500 c. 1 or more d. More than 20 Question 67 Marks: 1 A Provident Fund acquires its status of recognition by ___________. Choose one answer. a. Labour Ministry b. SEBI c. PF Commissioner

d. IT Authority Question 68 Marks: 1 An auditor of an Exempted Provident Fund can be: Choose one answer. a. A member of FPSB India b. A practicing Chartered Accountant c. An employee of the Employee Provident Fund Organization d. The auditor of the company, which is having the exempted provident fund Question 69 Marks: 1 No relief under section 89 of the Income Tax Act 1961 is admissible if taxable Gratuity is in respect of services rendered for less than ________ years. Choose one answer. a. 15 b. 5 c. 9 d. None of the above. Question 70 Marks: 1 Mr. Roy has been an employee of a public sector undertaking for the past 25 years and is retiring the next year. He is eligible for gratuity as per the provisions of the Payment of Gratuity Act, 1972. He hopes to invest the proceeds along with the PF proceeds, in order to fund his retirement. Prior to his wedding, Mr. Roy had nominated his mother to receive his gratuity. Is his wife eligible to receive the gratuity proceeds in the event of his death? Choose one answer.

a. All nominations made prior to the employee acquiring a family are invalid once an employee has acquired a family. b. As long as a valid nomination is in place, the payment of gratuity will be made only to such nominees. c. The payment of gratuity will always be according to the registered will, irrespective of the nominations. d. The information is not sufficient for giving an answer. Question 71 Marks: 1 Which of the following is the recommendation of recommendations of OASIS report? Choose one answer. a. Control of SEBI over mutual fund b. No investment in equity for retirement planning purpose c. asset management by competing professional pension fund managers d. Bank should Offer more interest rate on deposits by individual for retirement purpose Question 72 Marks: 1 Under Workmen compensation Act , the amount of compensation payable to a workman depends on Choose one answer. a. the nature of injury caused by accident b. the monthly wages of the workman concerned c. None of the above d. Both A & B Question 73 Marks: 1

U/s 10 of the Income Tax Act, Gratuity paid to a Government Employee or employees of local authorities is exempt upto a limit of ______________. Choose one answer. a. Rs. 350000 b. Rs. 300000 c. Rs. 1000000 d. None of the above Question 74 Marks: 1 Which of the following cannot be categorized under Fiduciary Responsibility of a financial planner towards his/her client? Choose one answer. a. Disclose all material facts b. Get the best returns on client's investments c. Serve the client's best interest d. Act in utmost good faith Question 75 Marks: 2 How many withdrawals under normal circumstances are allowed in Tier 1 _______ and Tier II _______________ . Choose one answer. a. No limit & No withdrawals b. 10 & 20 c. No withdrawals & No limit on withdrawals

d. 10 & No limit on withdrawals Question 76 Marks: 2 Sundar has been an employee of a public sector undertaking for the past 25 years and is retiring the next year. He is eligible for gratuity as per the provisions of the Payment of Gratuity Act, 1972. He hopes to invest the proceeds along with the PF proceeds, in order to fund his retirement. Sundar does not receive his gratuity 45 days after his retirement. What are his rights? Choose one answer. a. Sundar should receive his gratuity within 30 days after his retirement. The employer has to pay penal interest for delay in the payment of gratuity. b. Sundar should receive his gratuity within 60 days after his retirement. He should wait for his settlement. c. Sundar should receive his gratuity within 21 days after his retirement. The employer has to pay penal interest at 15% for delay in the payment of gratuity. d. There is no legally determined time frame for payment of gratuity. Sundar has legal remedies if payment is not made "within reasonable time." Question 77 Marks: 2 Which of the following is not prescribed for an entity to act as bank under the Banking Regulation Act, 1949? Choose one answer. a. Banking company means any company which transacts the business of banking b. Power to Reserve Bank to issue directive to banks to determine policy for advances c. Balance sheet is to be got audited from qualified auditors d. A banking company can create a charge upon any unpaid capital of the company

You might also like